Ask for the second question 2 The third question 1 2 , the first year of mathematics, thank you

Updated on educate 2024-05-03
13 answers
  1. Anonymous users2024-02-08

    Solution: Set the water use xm3 in March.

    2x+2 6+4 4+8(15 x 10)=44 solution: x=4 6,15 4=11m3

    2x+12+4(15﹣x﹣6)=44

    When the water consumption in March exceeds 6m3

    , but not more than 10m3

    , if the water consumption is xm in March, it will exceed 6m3 in April

    , but not more than 10m3

    When: no solution (abandonment).

    So in March 4m3 of water was used

    In April, 11 m3 of water was used

    Solution: (1) There are X people in the seventh grade and Y people in the eighth grade.

    First of all, because when purchasing tickets separately, the ticket price paid by the seventh grade is 15 yuan for the person and the ticket price paid by the eighth grade is 12 yuan.

    15 * x + = 1575, i.e. x + = 105x + y > x + 105, so the total number of people is more than 100 people.

    Since the total number of people is more than 100 people, the ticket ** is 9 yuan, and the ticket purchase together needs to pay 1080 yuan, there is x + y = 1080 9 = 120, that is, the total number of people is 120 people.

    2), = 15, i.e. y = 75, x = 120 - 75 = 45 (people).

    There are 45 students in the seventh grade and 75 students in the eighth grade.

    I wish you progress in your studies, and hopefully.

  2. Anonymous users2024-02-07

    Set up x people in the seventh grade, y people in the eighth grade, as known by the question, x is less than 50, y is greater than 50 less than 100, and 15x+15

    Assuming x+y is greater than 100, then.

    Calculate the equation to get x,y, then verify whether x is less than 50, y is greater than 50 less than 100, and then assume that x+y is less than 100, then.

    Calculate x,y to verify whether x is less than 50 and y is greater than 50 and less than 100, and verify that it meets the correct answer.

  3. Anonymous users2024-02-06

    1: Let the speed of B be x, and the speed of 12* will be 16 km/h.

    2: Let the machine be x x 4-((2 3)x) 4-((1 3)x) (4* The total number of machines is: 36 is planned to be 36 4, that is, 9 days to complete.

  4. Anonymous users2024-02-05

    1.Solution: Let the velocity of B be x, then the equation can be columned:

    x+12)*(6/60)=

    The solution is x=16km h

    2.Solution: If there are x sets of machines and are planned to be completed in y days, then the system of equations can be listed:

    4*y*(1-2/3)= *x /4)-1) -1)4y=x---2)

    Solution: x=36, y=9

  5. Anonymous users2024-02-04

    1.Solution: Let the velocity of B be x, and the equation according to the meaning of the problem is obtained:

    12+x)*(6/60)=

    Solution: x=16

    2.Solution: The original plan was planned to be completed in x days, according to the title:

    x-[(2 3) (1 x)+(1 3) (solution: x=9.)

    The machine has: 4x=4*9=36

  6. Anonymous users2024-02-03

    1.Let B's velocity be x, then (12+x)*, and the solution is x=16 km/h2If there are always x machines, then x 4 = 1 + 2 x 12 + x 18, and the solution is x = 36 sets, which was originally planned to take 9 days.

  7. Anonymous users2024-02-02

    1.Solution: Let the velocity of B be xkm h

    From the title: 12x(6 60)+6 60x=

    Solution: x=

  8. Anonymous users2024-02-01

    There are 3 types of simple parts, which are 2 4 8, 4 8 16, 5 10 20. Then find out how much each cost, and you will know the answer if you meet the requirements.

  9. Anonymous users2024-01-31

    Question 1: Root number 36 is less than root number 40 is less than root number 49

    So 6 is less than the root number 40 and less than 7

    So a+b=6+7=13

    Question 2: The square of 5 is equal to 25, and the root number 25 is less than the root number 26, so 5 is less than the root number 26 and the root number 2 is approximately equal to, so that the root number 2 is greater than.

    The root number five is approximately equal to it, so it is less than 1 minus one

  10. Anonymous users2024-01-30

    6^2=36

    36<40<49, so a=6,b=7,a+b=135= 25, 25< 26, so 5< 26 2 is approximately equal to, so 2>

    5 is approximately equal to >1, so 5-1>1

  11. Anonymous users2024-01-29

    The second question, 9, cannot be 4 because the sum of the two sides is greater than the third side

    The third question can be 11, as long as the sum of the two sides is greater than the third side, and the difference between the two sides is less than the third side.

  12. Anonymous users2024-01-28

    The other side must be 9cm

    Because it is a waist triangle, if two sides are 4cm and one is 9cm, the other side cannot form a triangle, and the other side must be "6!".

    Because the sum of the two sides of a triangle must be greater than the other!

  13. Anonymous users2024-01-27

    1, b and.

    2. Positive: cycle) 2 12 13 Negative: -1 7 -2 3

    Rational numbers: -1 7 cycles) 12 13 -2 3 irrational numbers: 2

Related questions
13 answers2024-05-03

Question 1: The original number is x in the tenth place, and y in the single place, so as to obtain the equation. >>>More

26 answers2024-05-03

1) The absolute value of 1/102-101 plus the absolute value of 1/103-102 is also reduced by 1/103 minus 101/101) absolute value. >>>More

24 answers2024-05-03

Question 1: There are two cases: remove the absolute value 2004-2004x=20042004x=0 >>>More

20 answers2024-05-03

1.The test is the condition for the binary equation to be true. >>>More

10 answers2024-05-03

Uh, the question indicates that there is a weak heart.